1. In Christ
    Joined
    30 Apr '07
    Moves
    172
    30 Apr '07 23:43
    http://s117.photobucket.com/albums/o51/Jirakon/?action=view&current=MissingPiece.jpg

  2. In Christ
    Joined
    30 Apr '07
    Moves
    172
    30 Apr '07 23:45
    Whoa, that's doesn't look right.
  3. In Christ
    Joined
    30 Apr '07
    Moves
    172
    30 Apr '07 23:46
    Okay, ignore the first board. The second is correct. Sorry, it's my first time posting an FEN board.
  4. In Christ
    Joined
    30 Apr '07
    Moves
    172
    01 May '07 01:36
    By the way, this came from Raymond Smullyan's The Chess Mysteries of Sherlock Holmes. It's full of great retrograde analysis problems. I'd recommend that book to any chess enthusiast.
  5. Joined
    15 Feb '07
    Moves
    667
    01 May '07 01:59
    All I can determine thus far is as follows.

    1) Seeing as black king is in check, it cannot be Black Queen or Rook.

    2) Last move was cxd8+ with promotion to rook. It's the only way the last move could have happened.

    3) I suspect it isn't a White Pawn or a Black Knight, but cannot fully disprove those yet. 2 pawn captures could make it a white pawn, and one of the knights might be an underpromoted pawn (like the rook).
  6. Standard memberDutch Defense
    Stealer of Souls
    Account suspended
    Joined
    16 Feb '07
    Moves
    119052
    01 May '07 02:29
    Originally posted by Jirakon
    http://s117.photobucket.com/albums/o51/Jirakon/?action=view¤t=MissingPiece.jpg

    [fen]position[2nR3K/pk1Rp1p1/p2p4/P1p5/1Pp5/2PP2P1/4P2P/n7] [/fen]
    If you want to get the correct board each time, go to New Game, click on Advanced Options, click on the box that says play from set piece, set up the position, highlight the fen so that you can copy it, and paste it where it says Insert board form FEN.

    You're welcome.
  7. In Christ
    Joined
    30 Apr '07
    Moves
    172
    01 May '07 02:31
    A good start. If you want a hint, read on.








    The next thing you should determine is exactly what black piece did the white pawn just capture? As for your third point, there are seven white pawns currently on the board, and you just said that the eighth one promoted.
  8. In Christ
    Joined
    30 Apr '07
    Moves
    172
    01 May '07 02:32
    Oh, thanks Dutch Defense.
  9. Standard memberDeepThought
    Losing the Thread
    Quarantined World
    Joined
    27 Oct '04
    Moves
    87415
    01 May '07 14:5612 edits
    Apart from the doubled ones all the black pawns are on their original files, unless the d-pawn and trailing c-pawn swapped files by taking. The leading black c-pawn must have started life out as the f-pawn. To get there it must have taken three times and the leading a-pawn (starting from b7) took once. Each thing they took has to have been a piece (since white has 7 pawns and one just promoted) so black's pawns have taken exactly four white pieces. White is missing a total of 6 pieces, of which 4 have been taken on light squares. Two could have been taken on dark squares - or one on one of each colour if and only if the c and d pawns swapped round. If this has happenned (black takes with the c and d pawns so they swap files) then the missing piece cannot be a white one and the h-pawn cannot have promoted.

    To get where it did to promote the white pawn had to take at least 4 black pieces. There are two places it could have started from c2: when it has to take 3 times to get to c7 and once to promote, or from b2 when it has to take a total of 5 times. For the b and c pawns to swap files we would need white to take a total of 6 pieces which is only possible if black hasn't promoted or has and the promoted piece has been taken (one of the pieces taken in this scenario is the a - pawn now on the b-file so it would need to have taken a white piece). Assuming this then the pawn started on c2. The piece just taken cannot have been a rook or a queen as there is no piece that could have moved to uncover a check on the white king and it couldn't have got there without going from a square where white was in check. Therefore the piece taken was the dark squared black bishop. Conceivably black could have promoted the h-pawn to a dark squared bishop. Since we know that the piece that fell off cannot have been a black rook or queen and the light squared black bishop can't be on h4 the only black piece it can be is the a-pawn - provided that black can't have promoted.

    If the c and d pawns both took white pieces - then it can't be a white piece - black cannot have promoted (there's not enough pieces to take) so since it's not a major piece, a knight or the light squared bishop the piece that fell off must be the h pawn.

    So it remains to be shown either that the c and d pawns have swapped round - or that black hasn't promoted (black can't have - otherwise the problem is impossible).

    Edits: Sorting out logic...
  10. Standard memberDeepThought
    Losing the Thread
    Quarantined World
    Joined
    27 Oct '04
    Moves
    87415
    01 May '07 16:11
    In the second paragraph above I mean starting out from f2 or g2, not b2 and c2 ๐Ÿ˜ž
  11. In Christ
    Joined
    30 Apr '07
    Moves
    172
    01 May '07 18:15
    I'll give another hint in response to the last answer.

    Therefore the piece taken was the dark squared black bishop.

    That statement is not necessarily true. You say that the dark squared black bishop was able to get to d8. Look at e7 and g7, and you'll have to rethink that.

    Conceivably black could have promoted the h-pawn to a dark squared bishop

    If you still hold that a dark squared bishop was captured, then indeed Black must have done just that. By the same reasoning, then, there's no reason that the captured piece could not have been a knight.
  12. Standard memberSwissGambit
    Caninus Interruptus
    2014.05.01
    Joined
    11 Apr '07
    Moves
    92274
    02 May '07 00:131 edit
    Originally posted by Jirakon
    Whoa, that's doesn't look right.
    [fen]2nR3K/pk1Rp1p1/p2p4/P1p5/1Pp5/2PP2P1/4P2P/n7[/fen]
    White must have just played c7xN(B)d8=R+. This leads to two dark-square Bishops or three Knights for Black; he must have promoted. Pf7 is on c4, so Ph7 promoted via hxg2. Black made 5 pawn captures all on light squares; White's 6th missing piece, the dark-square Bc1, was not captured by pawns.

    To promote, White had to make at least 4 P captures, not counting Bf8 who died w/o moving. This leaves one missing Black piece unaccounted for.

    The piece on h4 can't be a bQ or bR (impossible check to both K's), nor a Bh4 (requires a 2nd Black promotion - impossible). The only piece left is a wB.
  13. Standard memberDeepThought
    Losing the Thread
    Quarantined World
    Joined
    27 Oct '04
    Moves
    87415
    02 May '07 15:201 edit
    Originally posted by SwissGambit
    White must have just played c7xN(B)d8=R+. This leads to two dark-square Bishops or three Knights for Black; he must have promoted. Pf7 is on c4, so Ph7 promoted via hxg2. Black made 5 pawn captures [b]all on light squares; White's 6th missing piece, the dark-square Bc1, was not captured by pawns.

    To promote, White had to make at least 4 P captures h K's), nor a Bh4 (requires a 2nd Black promotion - impossible). The only piece left is a wB.[/b]
    Apart from one possible loophole, I agree with your answer - the only candidate is the dark squared white bishop.

    We have to be certain that the h7 pawn went h7 ---> h3 and then took a piece on g2. Had the pawn that just promoted on d8 come from g2 then black's h - pawn could have taken a white piece on a dark square and still promoted. For this to be possible white would have to take with the pawn on f3, e4, d5 and c6 and finally on d8. That's 5 captures with the pawn, and the f7 bishop never moved for a total of 6 captures. For this to happen we need the f pawn has to take something on g3 to restore the final position and we've run out of things for it to take. So the pawn that became a rook started out on f2 and black must have taken on g2 - so your answer is correct.

    Jirakon: That was a good puzzle - thanks.
  14. Standard memberSwissGambit
    Caninus Interruptus
    2014.05.01
    Joined
    11 Apr '07
    Moves
    92274
    02 May '07 16:31
    Originally posted by DeepThought
    Apart from one possible loophole, I agree with your answer - the only candidate is the dark squared white bishop.

    We have to be certain that the h7 pawn went h7 ---> h3 and then took a piece on g2. Had the pawn that just promoted on d8 come from g2 then black's h - pawn could have taken a white piece on a dark square and still promoted. For this to ...[text shortened]... t have taken on g2 - so your answer is correct.

    Jirakon: That was a good puzzle - thanks.
    Yep. Black's ...hxg3 requires two extra wP captures, and we only have one to spare.
  15. Joined
    26 Jun '06
    Moves
    59283
    03 May '07 23:45
    Originally posted by Jirakon
    http://s117.photobucket.com/albums/o51/Jirakon/?action=view¤t=MissingPiece.jpg

    [fen]position[2nR3K/pk1Rp1p1/p2p4/P1p5/1Pp5/2PP2P1/4P2P/n7] [/fen]
    talk about a royal fork๐Ÿ˜‰
Back to Top

Cookies help us deliver our Services. By using our Services or clicking I agree, you agree to our use of cookies. Learn More.I Agree